You are on page 1of 37

1

Chapter 1 The properties of gases


Exercises
1.2(b) (a)Could 25 g of argon gas in a vessel of volume 1.5L exert a pressure of 2.0 bar at 30C if it
behaved as a perfect gas? If not, what pressure would it exert?(b) what pressure would it exert if it behaved
as a van der Waals gas?
Solution: (a) The perfect gas law is
nRT pV =
implying that the pressure would be
V
nRT
p =
All quantities on the right are given to us except n, which can be computed from the given mass
of Ar.
mol 6 62 0
gmol 95 39
g 25
1
.
.
n = =


so bar 5 10
L 5 1
273k) (30 ) mol Lbark 10 31 8 mol 626 0
-1 -1 2
.
.
. ( ) . (
p =
+
=


not 2.0 bar.
(b) The van der Waals equation is
2
m
m
V
a
b V
RT
p

=
so bar
mol
barK
p 4 . 10
) 6 62 . 0 / L 5 . 1 (
K ) 273 30 ( ) L 10 31 . 8 (
2
1 2
=
+
=



2
1.5(b) A sample of hydrogen gas was found to have a pressure of 125 kPa when the temperature was 23C,
what can its pressure be expected to be when the temperature is 11C?
Solution:The relation between pressure and temperature at constant volume can be derived from the perfect
gas law
nRT pV = so
f
f
i
T
p
Ti
p
T p = o and
The final pressure, then, ought to be
Pa
Pa
T
T p
p
i
f i
f
k 120
273)K (23
K ) 273 11 ( ) k 125 (
=
+
+
= =





3
1.7(b) The following data have been obtained for oxygen gas at 273.15K. Calculate the best value of the
gas constant R from them and the best value of the molar mass of O
2
.
P/atm 0.750 000 0.500 000 0.250 000
V
m
/Lmol
-1
29.8649 44.8090 89.6384
/(gL
-1
) 1.07144 0.714110 0.356975
Solution:
All gases are perfect in the limit of zero pressure. Therefore the extrapolated value of T pV
m
/ will give
the best value of R.
The molar mass is obtained from RT
M
m
nRT pV =
Which upon rearrangement gives
p
RT
p
RT

p
RT
V
m
M = =
The best value of M is obtained from an extrapolation of
p

versus p to p= 0the intercept is M/RT


Draw up the following table
) mol K atm L /( ) / ( /
-1 -1
T pV atm p
m
) atm L g /( ) / (
-1 -1
p
0.750 000 0.082 0014 1.428 59
0.500 000 0.082 0227 1.428 22
0.250 000 0.082 0414 1.427 90
From Fig. 1.1(a)
From Fig. 1.1(b)
1 - 1
0
atm L 42755 . 1
-
g
p
p
=
|
|
.
|

\
|
=

1
1 - 1 1 - 1 -
0
mol g 9987 . 31
) atm L 42755 . 1 ( K) 15 . 273 ( ) mol K atm L 0615 082 . 0 (
-

=
=
=
|
|
.
|

\
|
= g
p
RT M
p

The value obtained for R deviates from the accepted value by 0.005 per cent. The error results from the fact
that only three data points are available and that a linear extrapolation was employed. The molar mass,
however, agrees exactly with the accepted value, probably because of compensating plotting errors.
THE PROPERTIES OF GASES

1 - 1 -
0
mol K atm L 0615 082 . 0 =
|
.
|

\
|
= p
m
T
pV
4
1.8(b) At 100C and 120 Torr, the mass density of phosphorus vapour is 0.6388 Kgm
-3
. What is the
molecular formula of phosphorus under these conditions?
Solution:The mass density is related to the molar volume V
m
by

M
V
m

Where Mis the molar mass. Putting this relation into the perfect gas law yields
RT
pM
so RT pV
m
=

=
Rearranging this result gives an expression for M; once we know the molar mass, we can divide by
the molar mass of phosphorus atoms to determine the number of atoms per gas molecule
1
-1 -1 -1
gmol 124
Torr 120
) (0.6388gL K] ) 273 100 [( mol K LTorr 364 . 62

=
+
=

=
p
RT
M
The number of atoms per molecule is
00 . 4
gmol 0 . 31
gmol 124
1
1
=




5
1.10(b) A gas mixture consists of 320 mg of methane, 175 mg of argon, and 225 mg of neon. The partial
pressure of neon at 300K is 66.5 Torr. Calculate(a) the volume and (b) the total pressure of the mixture.
Solution: (a) The volume occupied by each gas is the same, since each completely fills the container.
Thus solving for V from eqn 14 we have (assuming a perfect gas)
L 3.14 L 7 13 . 3
66.5Torr
K) 300 ( ) mol K LTorr 36 . 62 ( mol) 10 5 11 . 1 (
K 300 Torr, 5 . 66 mol, 10 5 11 . 1
20.18gmol
g 225 . 0
n
1 - 1 - 2
Ne
2
1 -
Ne
=

=
= = =
= =

V
T p
p
RT n
V
J
J

(b) The total pressure is determined from the total amount of gas, n=n
CH4
+n
Ar
+n
Ne.


Torr 212
L 7 3.13
K) 300 ( ) mol K LTorr 36 . 62 ( mol) 10 8 3.54 (
[1] p
mol 10 8 3.54 mol 10 ) 5 11 . 1 438 . 0 5 1.99 (
mol 10 38 . 4
mol g 39.95
g 0.175
mol 10 5 1.99
mol g 16.04
g 0.320
1 - 1 - 2
2 2
3
1
2 -
1 4
=

= =
= + + =
= = = =


V
RT n
n
n n
J
Ar CH
6
1.13(b) Determine the ratios of (a) the mean speeds, (b)the mean kinetic energies of He atoms and Hg
atoms at 25C.
Solution:(a) The mean speed of a gas molecule is
079 . 7
003 . 4
59 . 200
) He (
) Hg (
) Hg (
) He (

8

2
1
2
1
2
1
=
|
.
|

\
|
=
|
|
.
|

\
|
=
|
.
|

\
|
t
=
M
M
c
c
so
M
RT
c

(b) The mean kinetic energy of a gas molecule is
2
2
1
mc ,where c is the root mean square speed
2
1
3

|
.
|

\
|
=
M
RT
c
So
2
2
1
mc is independent of mass, and the ratio of mean kinetic energies of He and Hg is 1.
7
1.14(b) The best laboratory vacuum pump can generate a vacuum of about 1 nTorr. At 25C and assuming
that air consists of N
2
molecules with a collision diameter of 395 pm, calculate (a) the mean speed of the
molecules, (b) the mean free path, (c) the collision frequency in the gas.
Solution: (a) The mean speed can be calculated from the formula derived in Example 1.6

2
1
8

|
.
|

\
|
t
=
M
RT
c =
1 2
2
1
1 - 3
1 - 1 -
ms 10 75 . 4
mol kg 10 02 . 28 (
K) 298 ( ) mol JK 314 . 8 8

=
|
|
.
|

\
|
t

=

c
(b) The mean free path is calculated from ] 33 [
2
2
1
p
kT
o
=
With
2 19 2 10 2
m 10 90 . 4 ) m 10 95 . 3 (

= t = t = o d
Then,
m 10 4
atm 1
Pa 10 1.013
Torr 760
atm 1
Torr) 10 1 ( ) m 10 90 . 4 ( 2
K) 298 ( ) JK 10 381 . 1

4
5
9 2 19
2
1
-1 23
=
|
|
.
|

\
|

|
.
|

\
|


=


(c) The collision frequency could be calculated from eqn 31, but is most easily obtained from eqn 32, since
and c have already been calculated
1 2
4
s 10 1
m 10 46 . 4
75 . 4


=

=
c c
z
Thus there are 100s between collisions, which is a very long time compared to the usual timescale
apparatus used to generate the very low pressure.

8
1.16(b) At an altitude of 15 km the temperature is 217 K and the pressure 12.1 kPa. What is the mean free
path of N
2
molecules? (=0.43nm
2
)
Solution: The mean free path is

| |
m 10 1 . 4
) atm Pa 10 1 . 12 ( m) (10 0.43 2
) (217K ) JK 10 381 . 1
2
7
1 3 2 9
2
1
1 23
2
1



=


o
=

p
kT

1.17(b) How many collisions per second does an N
2
molecule make at an altitude of 15 km?(See Exercise
1.16b for data.)
Solution: Obtain data from Exercise 1.17(a) is p
mkT
z o
|
.
|

\
|
t
=
2
1
6 1

Substituting K 217 ), u 02 . 28 ( , Pa 10 1 . 12 , nm 43 . 0
3 2
= = = = o andT m p
we obtain
( ) | |
2
1
1 23 27
3 2 18
) K 217 ( K J 10 381 . 1 kg) 10 6605 . 1 ( ) 02 . 28 (
) Pa 10 1 . 12 ( ) m 10 43 . 0 ( 4
t

=

z
=9.910
8
s
-1



9
1.21(b) Estimate the critical constants of a gas with van der Waals parameters a=1.32 atmL2mol
-2
and
b=0.0436Lmol
-1
.
Solution: The critical constants of a van der Waals gas are

K 109
) mol L (0.0436 ) K atm L 27(0.08206
) mol 8(1.32atmL
27
8
and
atm 7 25
K atm 08206L 0 27
mol L atm 32 1
27
Lmol 131 0 Lmol 0436 0 3 3
1 1
2 2
2 1
2 - 2
2
1 1
c
=

= =
= = =
= = =


Rb
a
T
.
) . (
.
b
a
p
. ) . ( b V
c
c












1.22(b) A gas at 350K and 12 atm has a molar volume 12 per cent larger than calculated from the perfect
gas law. Calculate (a) the compression factor under these conditions and (b) the molar volume of the gas.
Which are dominating in the sample, the attractive or the repulsive forces?
Solution: The compression factor is


mol L 7 2
atm 12
K) (350 ) mol K atm L (0.08206
(1.12)
(1.12) (1.12)
is volume molar The (b)
dominate. forces Repulsive 12 1
have we 0.12 12 0 Beacuse (a)
1
1 - 1 -
m

=
|
|
.
|

\
|
=
|
|
.
|

\
|
= =
=
= + =
= =
. V
p
RT
V V
. Z
V V . V V
V
V
RT
pV
Z
perfect . m
perfect . m perfect . m perfect . m
perfect . m
m m


10
1.24(b) The density of water vapour at 1.00 bar and 383 K is 0.5678 kg m
-3
.(a) Determine the molar
volume V
m
of water and the compression factor Z, from these data. (b) Calculate Z from the van der Waals
equation with a=5.536L
2
atm mol
-2
and b=0.03049L mol
-1
.
Solution: (a)
4 995 0
mol 03049L 0 mol L 8 31.72
mol atm 536L 5
mol L 03049 0 mol L 728 31
mol L 8 31.72

-
get we above for expression the into ing subsititut and p Using (b)
0.9963
K) (383 ) mol K bar L (0.083145
) mol L 8 (31.72 bar) (1.00

mol L 8 72 31
mol g 5678 0
mol g 015 18

1 - 1 -
2 2
1 - 1 -
1 -
2
1 1
1
1
1
1
.
.
.
. .
RT V
a
b V
V
Z
Z
V
a
b V
RT
RT
pVm
Z
.
.
.

M
V
m m
m
m
m
m
=

=
=

= =
= = =


Comment. Both values of Z are very close to the perfect gas value of 1.000, indicating that water vapour is
essentially perfect at 1.00 bar pressure.

11
1.25(b) At 300 K and 20 atm, the compression factor of a gas is 0.86. Calculate (a) the volume occupied by
3.2 mmol of the gas under these conditions and (b) an approximate value of the second virrial coefficient B
at 300 K.
Solution:
| |
( )
( )
1 - 1 -
3 - 1 - 3 -
1
1
mol L 15 0 1 86 0 ) mol L 59 (1.0
1 1
expansion series the of truncation by 36 eqn from obtained be can of value e approximat An (b)
mL 8.7 L 10 8.7 ) mol L 59 (1.0 mol) 10 (8.2 Then, (a)
mol L 59 0 1
atm 20
K) (300 ) K atm L 0.08206

yields which for 34 solving by obtained is volume molar The
. .
Z- V
RT
pV
V B
B
nV V
.
p
ZRT
V
V ,
RT
pV
Z
m
m
m
m
m
m
m
= =
=
|
.
|

\
|
=
= = = =
=

= =
=



12
1.27(b) The critical constants of ethane are p
c
=45.6 atm, V
c
=148 cm
3
mol
-1
, and T
c
=305.4 K. Calculate the
van der Waals parameters of the gas and estimate the radius of the molecules.
Solution:
( )
( )
( ) ( )
( )
( )
be. should
it than lower cent per 25 about is computed our suggesting K, 305.4 is reported the However,
K 231
) mol L (0.0493 mol K atm L 0.08206 27
mol atm L 3.16 8
be should , determined already have we constants the to According
27
8
n informatio of piece another have We ined. overdeterm is problem this But
mol atm L 16 3 mol L 0493 0 atm 20 48 27 27 so
27
is pressure critical The
m 10 1.94 cm 10 347 1
) mol 10 4(6.02
) mol cm 3(49.3
2
1
4
3
2
1
so
3
2 4
volume excluded molar the is
constant Avogadro the times volume that radius); their twice (i.e., particles spherical those of diameter
the is radius whose sphere from excluded are particles spherical of centres The size. molecular of
estimate an obtain can we molecules, spherical of mole a of volume cxcluded the as ng interpreti By
mol L 0493 0 mol 3cm 49 mol cm 148
3
1
3
1
b so
3
is gas Waals der van a of volume critical The
1 1 1
2 2
2 2
2
1 2
2
10 - 8
3
1
1 23
1 3
3
1
3
1 1 3 1 3
a/b T
T
T
Rb
a
T
. . . b p a
b
a
p
. r
N
b
r
r
N b
b
a
b
. . V
b V
c
c
c
c
c
c
A
A
c
c
= =
=
= = =
=
= =
|
|
.
|

\
|

=
|
|
.
|

\
|
=
|
|
.
|

\
|
=
= = = =
=



13
1.29(b) Suggest the pressure and temperature at which 1.0 mol of (a) H
2
S, (b)CO
2
, (c) Ar will be in states
that correspond to 1.0 mol N
2
at 1.0 atm and 25C.
Solution:
K 56 3 (150.72K) (2.36)
atm 4 1 (48.00atm) (0.030)
Ar For (c)
K 718 (304.2K) (2.36)
atm 2.2 atm) (72.85 (0.030)
CO (b)For
) from obtained S H of constants (Critical
K 881 K) (373.2 (2.36)
atm 2.6 atm) (88.3 (0.030)
S H For (a)
are states ing correspond The
36 2
K 126.3
K 273) (25
and 030 0
atm 33.54
atm 1.0
are C 25 and atm 1.0 at N2 for e temperatur and pressure reduced
The . correspond to said are volume and e, temperatur pressure, reduced same the have that States
2
2
2
= = =
= = =
= = =
= = =
= = =
= = =
=
+
= = = = =

c r
c r
c r
c r
c r
c r
r
c
r
T T T
. p p p
T T T
p p p
ics. y and phys f chemistr handbook o
T T T
p p p
.
Tc
T
T .
p
p
p


14
1.30(b) A certain gas obeys the van der Waals equation with a=0.76 m
6
Pa mol
-2
. Its volume is found to be
4.0010
-4
m
3
mol
-1
at 288 K and 4.0Mpa. From this information calculate the van der Waals constant b.
What is the compression factor for this gas at the prevailing temperature and pressure?
Solution:
( ) ( )
( ) ( )
0.67
K 288 mol K J 8.3145
mol m 10 4.00 Pa 10 4.0
is factor n compressio The
mol m 10 .00 4
for solved be can which
is equation Waals der van The
1 1
3 4 6
1 3 4
2
2
1
=


= =
=
+
=

RT
pV
Z
V
a
p
RT
V b
b
V
a
b V
RT
p
m
m
m
m
m

15
Problems
Numerical problems
1.4 A meterological balloon had a radius of 1.0 m when released at sea level at 20C and expanded to a
radius of 3.0 m when it had risen to its maximum altitude where the temperature was -20C. What is the
pressure inside the balloon at that altitude?
Solution:
( )
( )
( ) atm 10 3.2 atm 1.0
K 293
K 253
3.0m
1.0m


3
4
3
4

3
4
and
3
4
ng Substituti
yields altitude, maximum its at pressure the , for Solving
constant, n with that, implies [12]
2
3
3
3
3
3 3

=
|
.
|

\
|

|
.
|

\
|
=

|
|
.
|

\
|
=
|
|
.
|

\
|
t
t
=
t = t =
=
= =
i
i
f
f
i
i
i
f
f
i
f
f f i i
i
i
f
f
i
f f
i
i i
f
f f
p
T
T
r
r
p
T
T
r
r
p
r V r V
p
T
T
V
V
p p
T
V p
T
V p
nRT pV


16
1.10 A vessel of volume 22.4 L contains 2.0 mol H
2
, and 1.0 mol N
2
at 273.15 K initially. All the H
2
reacted
with sufficient N
2
to form NH
3
. Calculate the partial pressure and the total pressure of the final mixture.4
Solution:
table following the up draw can We
2NH 3H N
is equation balanced The . completion to gone has reaction the after remains H2 no that assume We
3 2 2
+

N2 H2 NH3 Total
Initial amount n ' n 0 ' n n+
Final amount
' n n
3
1

0
' n
3
2
' n n
3
1
+
Specifically 0.33mol 0 1.33 mol 1.66 mol
Mole fraction 0.20 0 0.80 1.00
( )
( ) ( )
( ) ( )
( ) ( ) atm 0.33 atm 1.66 0.80 ) (NH ) (NH
atm 0.33 atm 1.66 0.20 ) (N ) (N
0 ) (H ) (H
atm 6 6 . 1
L 22.4
K 273.15 mol K atm L 10 8.206
1.66mol
3 3
2 2
2 2
1 1 2
= = =
= = =
= =
=
|
|
.
|

\
|
= =

p x p
p x p
p x p
V
nRT
p

17
1.15 Calculate the molar volume of chlorine gas at 350 K and 2.30 atm using (a) the perfect gas law and (b)
the van der Waals equation. Use the answer to (a) to calculate a first approximation to the correction term
for attraction and then use successive approximations to obtain a numerical answer for part(b).
Solution:
| | | |
( )
( )
. terminated be may ion approximat of cycle the so mol L 3 12
in results again expression first the of r denominato the into mol L 12.3 of on Substituti
mol L 3 12 mol L 10 622 5
2.34
mol L 2 7 28
mol L 10 622 5
) mol L (12.5
mol atm 6.579L
atm) (2.30
K) )(350 mol K atm L 10 (8.206
1.6 Table from b and a with Then
9b rearrange3 obtain we , 39b (b)From
mol L 5 12
atm 2.30
K) )(350 mol K atm L 10 (8.206
(a)
1
1
1 1 2
1
1 2
2 1
2 2
1 1 2
2
2
1
1 1 2
, . V
. .
.
. V
,
b
V
a
p
RT
V
V
a
b V
RT
p
.
p
RT
V
m
m
m
m
m
m
m


=
~ + ~
+
|
|
.
|

\
|
+

~
+
|
|
.
|

\
|
+
=

=
=

= =






18
Chapter 2 The First Lawthe concepts
Exercises
2.4 (b) A sample consisting of 2.00 mol He is expanded isothermally at 22 from 22.8 L to 31.7 L (a)
reversibly, (b) against a constant external pressure equal to the final pressure of the gas, and (c)
freely(against zero external pressure). For the three processes calculate q,w,U,and H.
SolutionFor a perfect gas at constant temperature
-w q U = = A so 0
For a perfect gas at constant temperature, H A is also zero
) d( d pV U H + =
we have already noted that U does not change at constant temperature; nor does pV if the gas obeys
Boyles law. These apply to all three cases below.
(a) Isothermal reversible expansion
J 10 62 1
10 62 1
L 22.8
L 31.7
ln K 273) (22 ) mol K (8.3145J mol) -(2.00
ln
3
3 1 - 1 -
= =
= + =
=
. w q
.
V
V
nRT w
i
f

(b) Expansion against a constant external pressure
V p w
ex
A =
Where
ex
p in this case can be computed from the perfect gas law
J 10 38 1
J 10 38 1
m L 1000
22.8)L - (31.7 Pa) 10 (1.55 -
and
Pa 10 1.55 ) Lm 1000 (
L 31.7
K 273) (22 ) mol K (8.3145J mol) (2.00
p
3
3
3
5
5 3
1 - 1 -
= =
=

=
=
+
=
=

. w q
. w
so
nRT pV
(c)
Free expansion is expansion against no force, so 0 0 -w q , and w = = = as well.
19
2.6(b) A sample of argon of mass 6.56 g occupies 18.5 L at 305 K. (a) Calculate the work done when the
gas expands isothermally against a constant external pressure of 7.7 kPa until its volume has increased by
2.5 L. (b) Calculate the work that would be done if the same expansion occurred reversibly.
Solution :

( )
( ) ( )
( ) ( )
( )
J -52.8
L 18.5
L 18.5 2.5
ln K 305 mol K J 3145 8
mol g 39.95
g 6.56
-
ln (b)
J 19
m L 1000
2.5L Pa 10 7.7
1 - 1 -
1 -
3
3
=
+

|
|
.
|

\
|
=
=
=

= A =

.
V
V
-nRT w
V p w a
i
f
ex


20
2.8(b) A sample of 2.00 mol CH
3
OH(g) is condensed isothermally and reversibly to liquid at 64. The
standard enthalpy of vaporization of methanol at 64 is 35.3 kJ mol
-1
. Find q,w,U,and H for this
process.
Solution: ( ) ( ) kJ 6 70 mol J 3K 35 mol 2.00 ) (
1
vap
. . H n H q = = = =


Because the condensation also occurs at constant pressure, the work is
V p V p w
ex
A = =
}
d
The change in volume from a gas to a condensed phase is approximately equal in magnitude to the volume
of the gas
( ) ( ) ( )
kJ 0 65 kJ 60 5 6 70
J 10 60 5 K 273 64 mol K kJ 3145 8 00mol 2 V -
3 -1 -1
vapour
. - ) . . (- w q U
. . . nRT p w
= + = + =
= + = = ~

21
2.11(b) The constant-pressure heat capacity of a sample of a perfect gas was found to vary with temperature
according to the expression C
p,m
/(JK
-1
)=20.17+0.4001(T/K). Calculate q,w,U,and H for 1.00 mol when
the temperature of 1.00 mol of gas is raised from 0 to 100 (a) at constant pressure (b) at constant
volume.
Solution
| |
( ) ( ) ( )
( ) ( ) ( ) ( )
( ) ( ) ( )
( )
kJ 1 14 so and 0 volume, constant At above. as
kJ 1 14 and kJ 9 14 Thus, gases. perfect in e temperatur on only depend and (b)
kJ 14.1 kJ 0.831 - 14.9
J 831 100 mol 3145J 8 mol 00 1
J 10 9 14 J 273 373 4001 0
2
1
273 373 17 20
K J K T 4001 0
2
1
T 17 20
K J dT K T 4001 0 17 20 CpdT q
pressure constant (a)At
1 1
3 2 2
373K
273K
1 2
273K 100
273K 0
1
. q , q U w
. U . H H U
w q U
K . . nRR p w
H . . .
/ . .
/ ) . ( .
+ = = =
= =
= = + =
= = = =
A = =
(

+ =
(

+ =
+ = =

+
+

} }



22
2.13(b) A sample of nitrogen of mass 3.12 g at 23.0 is allowed to expand reversibly and adiabatically
from 400 mL to 2.00 L. What is the work done by the gas?
Solution: Reversible adiabatic work is
( ) ( )
i f m , p V
T T R C n T C w = A =
Where the temperatures are related by [solution to Exercise2.12b]
( ) | |
( ) ( ) J 325 K 296 156 mol K J 8.3145 29.125
mol g 28.0
g 3.12
and
K 156
L 2.00
L 10 400
K 273.15 23.0 T So
503 2
mol K J 8.3145
mol K J 8.3145) (29.125
where
1 1
1 -
2.503 1
3
f
1 1
1 1
1
=
|
|
.
|

\
|
=
=
|
|
.
|

\
|
+ =
=

= =
|
|
.
|

\
|
=



w
.
R
R C
R
C
c
V
V
T Tf
p,m v,m
c
f
i
i

23
2.15(b) Calculate the final pressure of a sample of water vapour of mass 1.4 g that expands reversibly and
adiabatically from an initial temperature of 300 K and volume 1.0 L to a final volume of 3.0 L. Take =1.3.
Solution: For reversible adiabatic expansion


|
|
.
|

\
|
= =
f
i
i f i i f f
V
V
p p V p v p so
We need
i
p , which we can obtain from the perfect gas law
( ) ( )
( ) atm 0.46
L 3.0
L 1.0
atm 1.9
atm 1.9
L 1.0
K 300 mol K atm L 0.08206
mol g 18
g 1.4
so
1.3
1 1
1
=
|
.
|

\
|
=
=

|
|
.
|

\
|
=
= =

f
i
p
p
V
nRT
p nRT pV

24
2.19(b) When 2.0 mol CO
2
is heated at a constant pressure of 1.25 atm, its temperature increases from 250
K to 277 K. Given that the molar heat capacity of CO
2
at constant pressure is 37.11 JK
-1
mol
-1
, calculate
q,U,and H .
Solution:
( ) ( ) ( )
( )
( ) ( ) ( )
1 3 1 1 1 3
1 3
1 1
mol J 10 1.6 K 250 277 mol K J 8.3145 mol 2.0 mol J 10 2.0
so
mol J 10 2.0
K 250 277 mol K J 37.11 mol 2.0


= =
= + = + =
=
= = = =
U
nRR H U nRR U pV U H
T nC T C q H
p,m p p

25
2.21 (b) A sample consisting of 2.5 mol of perfect gas at 220 K and 200 kPa is compressed reversibly and
adiabatically until the temperature reaches 255 K. Given that its molar constant-volume heat capacity is
27.6 JK
-1
mol-1, calculate q,w,U,and H,and the final pressure and volume.
Solution: For adiabatic compression, q=0 and
( ) ( ) ( )
( )
( ) ( ) ( ) J 10 3.1 K 220 255 mol K J 8.3145 mol 2.5 J 10 2.4
J 10 2.4
J 10 2.4 K 220 255 mol J K 27.6 mol 2.5
3 1 - 1 3
3
3 1 1
= + =
+ = + =
= + =
= = A =


nRR U pV U H
w q U
T C w
V

The initial and final states are related by
( )
Pa 10 8 3
m 0.014
K 255 mol K J 8.3145 mol 2.5
L 14 m 0.014
K 255
K 220
m 9 0.022
m 9 0.022
Pa 10 200
K 220 mol K J 8.3145 mol 2.5
3.32
mol K J 8.314
mol K J 27.6
where
so
5
3
1 1
3
3.32
3
3
3
1 1
1 1
1 1 -
=

= =
= =
|
.
|

\
|
=
=


= =
= = =
|
|
.
|

\
|
= =


.
V
nRT
p
V
p
nRT
V
R
C
c
T
T
V V T V T V
f
f
f
f
i
i
V,m
c
f
i
f
c
i
c
f f
i
i i

26
2.24(b) Consider a system consisting of 3.0 mol O
2
(assumed to be a perfect gas) at 25 confined to a
cylinder of cross-section 22 cm
2
at 820 kPa. The gas is allowed to expand adiabatically and irreversibly
against a constant pressure of 1.0 atm. Calculate q,w,U,and H and T when the piston has moved 15
cm.
Solution: In an adiabatic process, q=0. Work against a constant external pressure is
( )
( ) ( )
( )
( )
( ) ( )
( )
( ) ( ) J 50 K 0.57 ) mol K J (8.3145 mol 3.0 J 36
H
K 0.57
mol K J 3145 8 355 29 mol 3.0
J 36
so
J 36
J 36
) m cm (100
cm 22 cm 15
Pa 10 110
1 - 1 -
1 - 1 -
3 1
2
3
= + =
+ = + = A
=


=
=
= =
= + =
=

= A =

nRR U pV U
. .
-R C n
w
T
T -R C n T C w
w q U
V p w
p,m
p,m V
ex


27
2.27(b) The standard enthalpy of formation of phenol -165.0 kJ mol
-1
. Calculate its standard enthalpy of
combustion.
Solution: The reaction is
( ) ( ) ( ) ( )
( ) ( ) ( ) ( ) | |
1
1
2 5 6 2 c
2 2 2 5 6
mol kJ 3053.6
mol J K 0 7 0 165 83 285 3 51 393 6
O 7 OH H C O H 3 CO2 6 H
O 3H 6CO 7O OH H C

O O O O O
=
+ =
A A A + A = A
+ +
. . .
H H H H
f f f f


28
2.29(b) From the following data, determine

f
H for diborane, B
2
H
6
(g), at 298 K:
( ) ( ) ( ) ( ) ( )
( ) ( ) ( ) ( )
( ) ( ) ( ) ( )
1
r 2 2 2
1
r 3 2 2
1
r 2 3 2 2 6 2
mol kJ 8 241 g O H g O
2
1
g H 3
mol kJ 2368 s O B g O
2
3
s 2B 2
mol kJ 1941 g O 3H s O B g 3O g H B 1

= A +
= A +
= A + +
. H
H
H


Solution: We need
O
A H
f
for the reaction
( ) ( ) ( ) ( )
{ } { } { }
( ) ( ) { }
1 1
6 2 2
mol J K 1152 mol J K 1941 8 241 3 2368
) reaction(1 ) reaction(3 3 ) reaction(2 Thus,
) reaction(1 - ) reaction(3 3 ) reaction(2 ) reaction(4
g H B g 3H s 2B 4

O O O O
= + =
A A + A = A
+ =
+
.
H H H H
r r r f


29
2.32(b) When 2.25 mg of anthracene, C
14
H
10
(s), was burned in a bomb calorimeter the temperature rose by
1.35 K. Calculate the calorimeter constant. By how much will the temperature rise when 135 mg of phenol,
C
6
H
5
OH(s), is burned in the calorimeter under the same conditions?
Solution: For anthracene the reaction is
( ) ( ) ( ) ( )
| |
( )
( ) ( ) ( ) ( )
( )
( ) ( ) ( )
( )
would. it
work precise very for but , of instead used had we if mattered have not would it figures,
t significan 3 within to Thus, cent. per 0.1 by differed and case this In Comment.
K 1 63
K kJ 0.0693
kJ 5 4.37
kJ 5 4.37 mol kJ 3050
mol g 94.12
g 10 135
mol kJ 3050
298 mol K kJ 10 314 8
2
3
mol kJ 3054
mol
2
3
-
Table2.5 mol kJ 3054
l O 3H g 6CO g O
2
15
s OH H C is reaction the used is phenol When
K J 69.3 K kJ 0.0693
K 1.35
kJ 0.0935
C
0.0935kJ
mol kJ 7157
mol g 172.23
g 10 2.25
mol kJ -7157
K) 298 (assume 298 mol K kJ 10 3 8
2
5
- mol kJ 7163
) ( mol kJ 7163
mol
2
5
- 26
l O 5H g 14CO g O
2
33
s H C
1
1
1
3
1
1 1 - 3 1
1
2 2 2 5 6
1 1
1
1
3
1
1 1 - 3 1
1
2 2 2 10 14

c
g g

c
g g

c
U H
H U
.
C
q
T
q
K .
n , RT n H U
H
T
q
U n qV q
T K . U
sics ry and phy of chemist Hand book H
n RT n H U
~
+ = = =
=
|
|
.
|

\
|
=
=

|
.
|

\
|
+ =
= A A =
=
+ +
= = =
A
=
=

|
|
.
|

\
|
= = =
=
=
|
.
|

\
|
=
=
= A A =
+ +


30
2.35(b) Given that the standard enthalpy combustion of graphite is -393.51 kJ mol
-1
and that of diamond is
-395.41kJ mol
-1
, calculate the enthalpy of the graphitediamond transition.
Solution: The difference of the equations is ( ) ( ) d C gr C
( ) | |
1 1
mol kJ 90 1 mol kJ 395.41 393.51

+ = = . H

trans

31
2.39(b) Use standard enthalpies of formation to calculate the standard enthalpies of the following reactions:
( ) ( ) ( )
( ) ( ) ( ) ( ) ( ) l O H aq NaCl aq NaOH aq HCl b
g propene g ne Cyclopropa a
2
+ +


Solution:
( ) ( ) ( ) ( ) | |
( )
( ) ( ) ( ) ( ) ( ) ( ) ( )
( ) ( ) ( )
( ) ( ) ( )
( ) ( ) ( ) | |
1
1
2
2
2
1
1
mol kJ 84 55
mol kJ 99 229 0 83 285
aq OH aq H l O, H
l O H aq OH aq H is and
l O H aq Cl aq Na aq OH aq Na aq Cl aq H
from obtained is reaction ionic met The
mol kJ 32.88
mol kJ 30 53 42 20

+
+
+ + +

=
=
=
+
+ + + + +
=
= =
.
. .
, H , H H H
b
. . g , e cycloropan H g , propene H H ) a (

r

32
2.40(b) Given the reactions(1) and (2) below, determine (a)

H
r
and

U
r
for reaction (3), (b)

f
H for both HI(g) and H2O(g) all at 298 K. Assume all gases are perfect.
( ) ( ) ( ) ( )
( ) ( ) ( ) ( )
( ) ( ) ( ) ( ) ( ) g O 2H s I g O g 4HI 3
mol kJ 64 483 g O 2H g O g 2H 2
mol kJ 96 52 g 2HI g I g H 1
2 2 2
1
r 2 2 2
1
r 2 2
+ +
= A +
+ = A +

. H
. H


Solution:
( ) ( ) ( ) ( )
( ) ( ) ( )
( )
( ) ( ) ( )
( ) ( ) ( )
( ) ( )
1 1
2
1 1
1 1 1
1 1 - 1
1
1 1
mol kJ 82 241 mol kJ 64 483
2
1
mol kJ 48 26 mol kJ 96 52
2
1
mol kJ 13 582 mol kJ 43 7 mol -589.56kJ
K 298 mol K J 314 8 3 mol -589.56kJ

mol -589.56kJ
mol kJ 96 52 2 mol kJ 64 483
1 2 2 3
1 reaction 2 2 reaction 3 reaction




= =
= =
= + =
=
A =
=
=
=
=
. . O H H
. . HI H b
. .
.
RT n H U
. .
H H H ) a (

f
g

r

33
2.44(b) Calculate

H
r
and

r
U at 298 K and

H
r
at 348 K for the hydrogenation of ethyne
(acetylene)to ethane (ethylene) from the enthalpy of combustion and heat capacity data in Tables 2.5 and
2.6. Assume the heat capacities to be constant over the temperature range involved.
Solution: The hydrogenation reaction is
( ) ( ) ( ) ( ) ( ) ? T
c
= +

r 4 2 2 2 2
H g H C g H g H C 1
The reactions and accompanying data which are to be combined in order to yield reaction (1) and
Are
( ) ( ) ( ) ( ) ( )
( ) ( ) ( ) ( ) ( ) ( )
( ) ( ) ( ) ( ) ( ) ( )
( ) ( ) ( ) ( ) ( )
( ) ( ) ( ) { }
( ) ( ) | |
( )
( ) ( ) ( ) ( )| |
( )| | ( ) ( ) ( )
( )
( ) ( ) ( ) ( )
1
1 1 - 3 1
r
1 1 - 3 1 1 - 3
2 2 2 4 2 r
r

r
1 1 1
1
1

r
1
C 2 2 2 2 2
1
C 2 2 2 4 2
1
C 2 2 2
mol kJ 176
K 50 mol K kJ 10 19 29 mol kJ 175 348 H
mol K kJ 10 19 29 mol K kJ 10 82 28 93 43 56 43
H H C H C 47 J C
Example2.7 K 298 K 348 C 298 H K 348 H b
mol kJ 173 mol kJ 48 2 mol 175kJ -
1 26
mol kJ 175
mol kJ 1300 1411 83 285
4 H 3 H 2 H H a
Hence,
) reaction(4 ) reaction(3 - ) reaction(2 ) reaction(1
mol kJ 1300 4 H g CO 2 l O H g O
2
5
g H C 4
mol kJ 1411 3 H g 2CO l O 2H g 3O g H C 3
mol kJ 285.83 2 H l O H g O
2
1
g H 2

=
=
= =
= =
=
= + =
= =

+ =
+ =
+ =
= + +
= + +
= +

. K
. . . .
g , C g , C g , C C v
K
.
- RT n T H T U
.
T
m , p m , p m , p
J
m , p J p
p
g g

r


34
Problems
2.2 An average human produces about 10MJ of heat each day through metabolic activity. If a human body
were an isolated system of mass 65 kg with the heat capacity of water, what temperature rise would the
body experience? Human bodies are actually open systems, and the main mechanism of heat loss is through
the evaporation of water. What mass of water should be evaporated each day to maintain constant
temperature?
Solution: Good approximate answers can be obtained from the data for the heat capacity and molar heat of
vaporization of water at 25.[Table 2.6 and 2.3]
( ) ( )
( )
( ) ( )
( ) ( )
kg 09 4
mol 44.0kJ
kJ 10 0 1 mol kg 018 0
H
H H From
K 37
mol K kJ 0.0753 mol 10 3.6
kJ 10 1.0
obtain we From
mol 10 6 3
mol kg 0.018
kg 65
O H
mol kJ 0 44 O H H mol K J 3 75
1
4 1
vap
vap vap
1 1 - 3
4
3
1
2
1
2 vap
1 1
2
.
. . H M
m
M
m
n H
nC
H
T
, T nC H
. n
. . O H C
p,m
p,m
m , p
=

=
A

=
A = A =
+ =


= =
=
= =
= A =

O
O O

O



Comment: This estimate would correspond to about 30 glasses of water per day, which is much higher than
the average consumption. The discrepancy may be a result of our assumption that evaporation of water is
the main mechanism of heat loss.



35
Chapter 3 The First Law: the machinery
Exersises
3.10(b) A vapour at 22 atm and 5 was allowed to expand adiabatically to a final pressure of 1.00 atm; the
temperature fell by 10 K. Calculate the Joule-Thomson coefficient, , at 5, assuming it remains constant
over this temperature range.
Solution:
The Joule-Thomson coefficient is the ratio of temperature change to pressure change under conditions
of isenthalpic expansion. So
( )
1
atm K 0.48
atm 22 1.00
K 10

=

= ~
|
|
.
|

\
|
c
c
=
p
T
p
T

H


36
3.11(b) For a van der Waals gas,
2
m
T
V
a
= . Calculate
m
U A for the isothermal reversible expansion of
argon from an initial volume of 1.00 L to 22.1 L at 298 K. What are the values of q and w?
Solution:
( )
( ) ( )
( ) ( )
( )
( )
( ) ( )
1 - 1 - 1 - 1 -
1 -
2
2
1 - 1 -
1 22
00 1
mol L 22.1
mol L 1.00
2
2
1 1 - 3
3
3
1 5
1 5 2 2
2 2 1
2 2
1 -
1 - 1 - 1 -
mol 1L 22
mol L 1.00 2
mol L 22.1
mol L 1.00
2
m2
m1
2
m2
m1
2
T
mol kJ 62 7 mol J 7 761 mol J 130 mol J 7 774
mol kJ 69 7.74
10 22 3 00 1
10 22 3 1 22
ln K 298 mol K J 8.314
Thus
d d so
gas Waals der van a for
mol J 130 mol am 130P
L 10
1m
atm Pa 10 01325 1
atm Pa 10 01325 1 mol L atm 41 28 0
mol L atm 1.345 L mol 95475 0
mol L atm 1.345
mol L 75 954 0
mol 1L 22
21.1
mol L 1.00 mol 1L 22
a

so process, isothermal an in 0

1
1
1 -
1 -
1 -
1 -
1 -
1 -
. w
. .
. .
b V ln RT dV
b V
RT
q
U q V
V
a
V
b V
RT
w
V
a
b V
RT
p and V d p w
.
. .
. U
a
.
.
a a
.
a
V
a
V
dV
dV
V
a
dU U
dV
V
a
dV
V
U
dU
dT
dV
V
U
dT
T
dU
dU T,V U U
-
-
m m
m
L mol .
L mol .
m m
m
m m
m
m
m
m
m
m ex
m
.
m m
m
V
V
m
m
V
V
m m
m
m
m
m
m
m
m
m
m
V
m
m m m m
= = + =
+ =
)
`



+ =
+ =
|
|
.
|

\
|

+ =
+ = +
|
|
.
|

\
|

= =
= =
|
|
.
|

\
|

=
= A
=
= = + =
= = = = A
=
|
|
.
|

\
|
c
c
=
=
|
|
.
|

\
|
c
c
+ |
.
|

\
|
c
= =

}
} }
}
} } }

37
3.12(b) The volume of a certain liquid varies with temperature as
( ) ( ) { }
2
6 4
T/K 10 52 1 T/K 10 7 3 77 0

+ + = . . . ' V V
Where ' V is its volume at 298 K. Calculate its expansion coefficient, o, at 310 K
Solution:
The expansion coefficient is
( )
( ) | |
( ) ( ) | |
( ) | |
( ) ( )
1 3
2
6 4
1 6 1 4
2
6 4
1 6 1 4
2 6 1 4
K 10 27 1
310 10 52 1 310 10 7 3 77 0
K 310 10 1.52 2 K 10 3.7
T/K 10 52 1 T/K 10 7 3 77 0
K T/K 10 1.52 2 K 10 3.7
K T 10 1.52 2 K 10 3.7 1






=
+ +
+
=
+ +
+
=
+
=
|
.
|

\
|
c
c
=
.
. . .
. . . ' V
V'
V
V'
T
V
V

You might also like